Difference between revisions of "User:John0512"

m (Undo revision 189989 by Ej22 (talk))
(Tag: Undo)
 
(10 intermediate revisions by 4 users not shown)
Line 1: Line 1:
==Orzorzorz Number==
+
==2021 AMC 8==
 +
2021 AMC 8 problems and solutions. The test has not been held, and will never be held.
  
This is a number I coined. The exact value of this number is <cmath>\sum_{m=17}^{122}\sum_{k=17}^{122}\sum_{n=17}^{\min(m,k)} (2^{24})^{n-17}\cdot(2^{730})^{\min(m,k)-n}\cdot26^{12m+12k},</cmath> and is approximately equal to <math>9.8\cdot10^{27216}</math>. The formula used to find this number is an approximation of the number of ways mathimathz can happen, however I will not go into further detail about exactly how it was derived. Each one of the numbers in the formula has a good reason, however I am not revealing just yet. :)
+
==Problems==
 +
ERROR: Content not found
 +
==Solutions==
 +
ERROR: Content not found
 +
==Results==
 +
 
 +
Highest Score: 0.00
 +
 
 +
Distinguished Honor Roll: 0.00
 +
 
 +
Honor Roll: 0.00
 +
 
 +
Average Score: 0.00
 +
 
 +
Standard Deviation: 0.00
  
 
==Unnamed Theorem==
 
==Unnamed Theorem==

Latest revision as of 03:03, 12 March 2023

2021 AMC 8

2021 AMC 8 problems and solutions. The test has not been held, and will never be held.

Problems

ERROR: Content not found

Solutions

ERROR: Content not found

Results

Highest Score: 0.00

Distinguished Honor Roll: 0.00

Honor Roll: 0.00

Average Score: 0.00

Standard Deviation: 0.00

Unnamed Theorem

I have something called the Unnamed Theorem (which I did not name as I have not confirmed that this theorem has not existed before).

Claim: Given a set $S=\{1,2,3\cdots n\}$ where $n$ is a positive integer, the number of ways to choose a subset of $S$ then permute said subset is $\lfloor n!\cdot e\rfloor.$

Proof: The number of ways to choose a subset of size $i$ and then permute it is $\binom{n}{i}\cdot i!$. Therefore, the number of ways to choose any subset of $S$ is \[\sum_{i=0}^n \binom{n}{i}\cdot i! = \sum_{i=0}^n \frac{n!}{(n-i)!}.\] This is also equal to $\sum_{i=0}^n \frac{n!}{i!}$ by symmetry across $i=\frac{n}{2}$. This is also $n! \cdot \sum_{i=0}^n \frac{1}{i!}.$ Note that $e$ is defined as $\sum_{i=0}^\infty \frac{1}{i!}$, so our expression becomes \[n!(e-\sum_{i={n+1}}^\infty \frac{1}{i!}).\] We claim that $\sum_{i={n+1}}^\infty \frac{1}{i!}<\frac{1}{n!}$ for all positive integers $n$.

Since the reciprocal of a factorial decreases faster than a geometric series, we have that $\sum_{i={n+1}}^\infty \frac{1}{i!}<\frac{1}{(n+1)!}+\frac{1}{(n+1)!(n+1)}+\frac{1}{(n+1)!(n+1)^2}\cdots$. The right side we can evaluate as $\frac{1}{n(n!)}$, which is always less than or equal to $\frac{1}{n!}$. This means that the terms being subtracted are always strictly less than $\frac{1}{n!}$, so we can simply write it as \[\lfloor n!\cdot e\rfloor.\]

Example: How many ways are there 5 distinct clones of mathicorn to each either accept or reject me, then for me to go through the ones that accepted me in some order?

Solution to example: This is equivalent to the Unnamed Theorem for $n=5$, so our answer is $\lfloor 120e \rfloor=\boxed{326}$.

Solution 2: Since I am not orz, all 5 clones will reject me, so the answer is $\boxed{1}$. Note that this contradicts with the answer given by the Unnamed Theorem.